LSAT and Law School Admissions Forum

Get expert LSAT preparation and law school admissions advice from PowerScore Test Preparation.

User avatar
 Dave Killoran
PowerScore Staff
  • PowerScore Staff
  • Posts: 5852
  • Joined: Mar 25, 2011
|
#43589
Complete Question Explanation
(The complete setup for this game can be found here: lsat/viewtopic.php?t=8007)

The correct answer choice is (B)

Answer choice (A) is incorrect because G and H cannot be selected together.

Answer choice (C) is incorrect because J requires W.

Answer choices (D) and (E) are both incorrect because H requires X.

Answer choice (B) is thus correct.

Get the most out of your LSAT Prep Plus subscription.

Analyze and track your performance with our Testing and Analytics Package.